You are on page 1of 6

UPSC Civil Services Main 1987 - Mathematics

Calculus
Sunder Lal
Retired Professor of Mathematics
Panjab University
Chandigarh

January 14, 2010

1 9 1 9
Question 1(a) If x1 = x+ and for n > 0, xn+1 = xn + , find the value of
2 x 2 xn
limn xn (x > 0 is assumed).

Solution. x > 0 x1r > 0. By induction, clearly xn > 0 for all n 1.


x + x9 9
Now x1 = x = 3, because the arithmetic mean is always the geometric
2 x
mean of two positive numbers. Similarly xn 3 for all n 1, thus the sequence {xn } is
bounded below. Moreover
1 9 1
xn+1 xn = xn + xn = (9 x2n ) 0
2 2xn 2xn
as x2n 9. Thus {xn } is a monotonically decreasing sequence. Let l be the greatest lower
bound of {xn } then
1 9  1 9  1 9
l = lim xn = lim xn1 + = lim xn1 + = l+
n n 2 xn1 2 n limn xn1 2 l
Thus 2l2 = l2 + 9 l = 3 (l = 3 is not admissible as all elements of the sequence are 3).
Thus limn xn = 3.
1
Question 1(b) 1. If x = a, h = 2a, f (x) = x 3 , find from the mean value theorem:
f (x + h) = f (x) + hf 0 (x + h).

2. If u = x + y z, v = x y + z, w = x2 + (y z)2 , examine whether or not there is a


functional relationship between u, v, w and find the relationship, if any.

Solution.

1
1. Substituting the given values, we get f (a) = f (a) + 2af 0 (a + 2a). Now f (x) =
1 2
x 3 f 0 (x) = 13 x 3 , so

1 12a 2
a 3 = (a) 3 + (a + 2a) 3
3
1 2a 2 2
= a 3 + a 3 (2 1) 3
3
2 2
2 = (2 1) 3
3
2
3 = (2 1) 3
27 = (2 1)2
1
= 2 1
27

So = 21 (1 1

3 3
).

2. To establish that u, v, w have a functional relation, we have to show that the Jacobian
(u, v, w)
= 0. Clearly
(x, y, z)

1 1 1 1 1 0
(u, v, w)
= 1 1 1 = 1
1 0 = 0
(x, y, z) 2x 2(y z) 2(y z) 2x 2(y z) 0

Thus u, v, w are functionally dependent.


Now u2 = x2 + (y z)2 + 2x(y z), v 2 = x2 + (y z)2 2x(y z), thus u2 + v 2 = 2w,
which is the desired functional relationship.

 x n1 +1 + y n1 +1  12
Question 1(c) If u = csc1 , show that
x+y

2u 2u 2
2 u 1
x2 2
+ 2xy + y 2
= 2 tan u(2n + sec2 u)
x x y y 4n
1
Solution. Let v = csc u. Then v is a homogeneous function of degree 2n
, because
 (ax) n1 +1 + (ay) n1 +1  12 1
 x n1 +1 + y n1 +1  12 1
v(ax, ay) = = a 2n = a 2n v(x, y)
a(x + y) x+y

2
Applying Eulers theorem to v,
v v 1
x +y = v
x y 2n
u u 1 v u
x csc u cot u y csc u cot u = csc u v = csc u, = csc u cot u
x y 2n x x
u u 1
x +y = tan u (1)
x y 2n
2 2
u u u 1 u
+x 2 +y = sec2 u Differentiating (1) w.r.t. x (2)
x x x y 2n x
2
u u 2u 1 u
x + +y 2 = sec2 u Differentiating (1) w.r.t. y (3)
x y y y 2n y
2 2
2u
 
u u 1 u u
x2 2 + 2xy + y2 2 = (1 2
sec u) x +y (2) x + (3) y
x x y y 2n x y
1 1
= (2n + sec2 u)( tan u) From (1)
2n 2n
1
= tan u(2n + sec2 u)
4n2

Question 2(a) 1. Show by means of a suitable substitution that




tx1
Z Z
2
2x1 2y1 1
sin cos d = dt, x, y > 0
0 2 0 (1 + t)x+y

2. Establish the inequality Z 1


1 dx
< 1 <
2 0 (4 x2 x3 ) 2 6

Solution.
Z 1
1. By definition B(y, x) = z y1 (1 z)x1 dz.
0
2
Put z = cos , dz = 2 cos sin d, to get
Z 0
B(y, x) = cos2y2 sin2x2 (2 cos sin ) d

2
Z
2
= 2 sin2x1 cos2y1 d
0

3
1
Now, put z = t+1
in B(y, x),
Z 1
B(y, x) = z y1 (1 z)x1 dz
0
Z 0
1 y1  1 x1 dt
= 1
t+1 t+1 (1 + t)2
Z
tx1
= dt
0 (1 + t)x+y

From these two we have


Z
1 tx1
Z
2
2x1 2y1
sin cos d = dt
0 2 0 (1 + t)x+y

as required.

2. Since 4 x2 + x3 = 4 x2 (1 x), and x2 (1 x) > 0 for 0 < x < 1, it follows that


1 1
4 x2 + x3 < 4 < 1 .
2 (4 x2 x3 ) 2
1 1
On the other hand, 4 x2 + x3 > 4 x2 for x > 0, so (4 x2 + x3 ) 2 < (4 x2 ) 2 .
Hence
Z 1 Z 1 Z 1
1 dx dx
dx < 1 <
0 2 4 x2
2 3
0 (4 x x ) 2 0
Z 1 1
1 dx 1 x
< 1 < sin =
2 2 3
0 (4 x x ) 2 2 0 6

x3
Question 2(b) Find the volume of the solid generated by revolving the curve y 2 = ,a >
2a x
0 about its asymptote x = 2a.

Solution. Consider a thin vertical strip of thickness dx, and rotate it about the asymptote

4
r
x3
its volume is 2rh dx = 2(2a x)2 dx. Thus
2a x
s
Z 2a
x3
V = 4 (2a x) dx
0 2a x

Z 2a
3
= 4 2a x x 2 dx
0
Put x = 2a sin2 dx = 4a sin cos d
Z
2 3
= 4 2a cos (2a) 2 sin3 4a sin cos d
0
Z
2
3
= 64a sin4 cos2 d
0
311
= 64a3 = 2 2 a3
642 2

ZZ
1 1 2
Question 2(c) Evaluate x 2 y 3 (1 x y) 3 dx dy where D is the domain bounded by the
D
lines x = 0, y = 0, x + y = 1.

Solution. We convert this to a Dirichlet integral using the standard transformation. Put
x + y = u, y = uv, so that dx dy = u du dv, (see 1989, question 3(a) for example).
Z 1Z 1
1 1 1 1 2
I = u 2 (1 v) 2 u 3 v 3 (1 u) 3 u du dv
Z0 1 Z0 1
11 2 1 1
= u 6 (1 u) 3 v 3 (1 v) 2 du dv
0 0
( 17
6
)( 53 ) ( 34 )( 32 ) 2
3
( 23 ) 31 ( 13 )( 32 ) 16  2   1 
= = =
( 276
) ( 17 6
) 753
222
( 32 ) 945 3 3

But ( 23 )( 13 ) =
sin 3
= 2

3
. Thus I = 32
.
945 3

Paper II

Question 3(a) Let f (x) = x if x is rational, and 1 x if x is irrational. Show that f is


continuous only at x = 12 .

Solution. See 2001, question 1(a).

5
Question 4(a) Find the maximum and minimum value of f (x, y) = xy subject to the con-
dition that x2 + y 2 + xy = a2 .

Solution. Let F (x, y) = xy (x2 + y 2 + xy a2 ), where is Lagranges undetermined


multiplier. For extreme values,
F F y x
= y 2x y = 0, = x 2y x = 0 = =
x y 2x + y 2y + x

as (x, y) 6= (0, 0). Thus 2x2 + xy = 2y 2 + xy (x + y)(x y) = 0 x = y, x = y. Using


x2 + y 2 + xy = a2 we get
a2
1. x = y 3x2 = a2 x = y = a3 f (x, y) = 3

2. x = y x2 = a2 x = a, y = a or x = a, y = a. In either of these cases


f (x, y) = a2 .
2
Thus the required maximum value is a3 and the required minimum is a2 .
Note: In this problem there was no need to check the nature of the critical points, as the
maximum amd minimum values occur at these points. If it were required, it could be done
as follows.
d2 F = 2(dx)2 2(dy)2 + 2(1 )dx dy
2x+y
Now x2 + y 2 + xy = a2 2x dx + x dy + y dx + 2y dy = 0, or dy = x+2y dx. Thus
 2
2 2 2x + y 2x + y
d F = 2(dx) 2 (dx)2 2(1 )(dx)2
x + 2y x + 2y

Case 1: x = a, y = a, = 1 or x = a, y = a, = 1

d2 F = 2(dx)2 4(1)(dx)2 + 2(1)2 (dx)2 > 0

so we have a local minimum at (a, a) or (a, a).


Case 2: x = y = a3 , = 31 .

2 4 2
d2 F = (dx)2 (dx)2 (dx)2 < 0
3 3 3
so we have a local maximum at x = y = a3 .

You might also like